Determinant of a Hankel matrix with sequence (1, 2, 3,…, n, 1, 2,…, n-1)












1












$begingroup$


I'm looking for a closed form of the determinant of matrices like



$begin{bmatrix}1 & 2 & 3\2 & 3 & 1\ 3& 1 &2end{bmatrix}$



or



$begin{bmatrix}1 & 2 & 3 &4\2 & 3 &4 & 1\ 3&4& 1 &2\ 4& 1&2&3end{bmatrix}$,



that means Henkel matrices of the size n with sequence (1, 2, 3,..., n, 1, 2,..., n-1)










share|cite|improve this question









$endgroup$












  • $begingroup$
    Have you tried something?
    $endgroup$
    – Tito Eliatron
    Jan 7 at 20:38










  • $begingroup$
    Yes, some row operations. I tried to create 1's in the first column by substracting row j from row j+1, but that didn't seem to help. I tried swapping the rows, such that the 1's are on the main diagonal, also not helpful I think.
    $endgroup$
    – Emphikongri
    Jan 7 at 20:43






  • 1




    $begingroup$
    Maybe you can add to the first row (or column) the others
    $endgroup$
    – Tito Eliatron
    Jan 7 at 20:44
















1












$begingroup$


I'm looking for a closed form of the determinant of matrices like



$begin{bmatrix}1 & 2 & 3\2 & 3 & 1\ 3& 1 &2end{bmatrix}$



or



$begin{bmatrix}1 & 2 & 3 &4\2 & 3 &4 & 1\ 3&4& 1 &2\ 4& 1&2&3end{bmatrix}$,



that means Henkel matrices of the size n with sequence (1, 2, 3,..., n, 1, 2,..., n-1)










share|cite|improve this question









$endgroup$












  • $begingroup$
    Have you tried something?
    $endgroup$
    – Tito Eliatron
    Jan 7 at 20:38










  • $begingroup$
    Yes, some row operations. I tried to create 1's in the first column by substracting row j from row j+1, but that didn't seem to help. I tried swapping the rows, such that the 1's are on the main diagonal, also not helpful I think.
    $endgroup$
    – Emphikongri
    Jan 7 at 20:43






  • 1




    $begingroup$
    Maybe you can add to the first row (or column) the others
    $endgroup$
    – Tito Eliatron
    Jan 7 at 20:44














1












1








1





$begingroup$


I'm looking for a closed form of the determinant of matrices like



$begin{bmatrix}1 & 2 & 3\2 & 3 & 1\ 3& 1 &2end{bmatrix}$



or



$begin{bmatrix}1 & 2 & 3 &4\2 & 3 &4 & 1\ 3&4& 1 &2\ 4& 1&2&3end{bmatrix}$,



that means Henkel matrices of the size n with sequence (1, 2, 3,..., n, 1, 2,..., n-1)










share|cite|improve this question









$endgroup$




I'm looking for a closed form of the determinant of matrices like



$begin{bmatrix}1 & 2 & 3\2 & 3 & 1\ 3& 1 &2end{bmatrix}$



or



$begin{bmatrix}1 & 2 & 3 &4\2 & 3 &4 & 1\ 3&4& 1 &2\ 4& 1&2&3end{bmatrix}$,



that means Henkel matrices of the size n with sequence (1, 2, 3,..., n, 1, 2,..., n-1)







matrices determinant integers






share|cite|improve this question













share|cite|improve this question











share|cite|improve this question




share|cite|improve this question










asked Jan 7 at 20:32









EmphikongriEmphikongri

83




83












  • $begingroup$
    Have you tried something?
    $endgroup$
    – Tito Eliatron
    Jan 7 at 20:38










  • $begingroup$
    Yes, some row operations. I tried to create 1's in the first column by substracting row j from row j+1, but that didn't seem to help. I tried swapping the rows, such that the 1's are on the main diagonal, also not helpful I think.
    $endgroup$
    – Emphikongri
    Jan 7 at 20:43






  • 1




    $begingroup$
    Maybe you can add to the first row (or column) the others
    $endgroup$
    – Tito Eliatron
    Jan 7 at 20:44


















  • $begingroup$
    Have you tried something?
    $endgroup$
    – Tito Eliatron
    Jan 7 at 20:38










  • $begingroup$
    Yes, some row operations. I tried to create 1's in the first column by substracting row j from row j+1, but that didn't seem to help. I tried swapping the rows, such that the 1's are on the main diagonal, also not helpful I think.
    $endgroup$
    – Emphikongri
    Jan 7 at 20:43






  • 1




    $begingroup$
    Maybe you can add to the first row (or column) the others
    $endgroup$
    – Tito Eliatron
    Jan 7 at 20:44
















$begingroup$
Have you tried something?
$endgroup$
– Tito Eliatron
Jan 7 at 20:38




$begingroup$
Have you tried something?
$endgroup$
– Tito Eliatron
Jan 7 at 20:38












$begingroup$
Yes, some row operations. I tried to create 1's in the first column by substracting row j from row j+1, but that didn't seem to help. I tried swapping the rows, such that the 1's are on the main diagonal, also not helpful I think.
$endgroup$
– Emphikongri
Jan 7 at 20:43




$begingroup$
Yes, some row operations. I tried to create 1's in the first column by substracting row j from row j+1, but that didn't seem to help. I tried swapping the rows, such that the 1's are on the main diagonal, also not helpful I think.
$endgroup$
– Emphikongri
Jan 7 at 20:43




1




1




$begingroup$
Maybe you can add to the first row (or column) the others
$endgroup$
– Tito Eliatron
Jan 7 at 20:44




$begingroup$
Maybe you can add to the first row (or column) the others
$endgroup$
– Tito Eliatron
Jan 7 at 20:44










2 Answers
2






active

oldest

votes


















2












$begingroup$

The magnitude of the determinant is $$frac{n^{n-1}(n+1)}{2}.$$ The sign is given by $$f(n) = begin{cases}+ & mathrm{floor}(n/2) text{ is even.} \ - & mathrm{floor}(n/2) text{ is odd} .end{cases}$$A useful formula is $sum_{i=1}^{n} i = frac{n(n+1)}{2}$.






share|cite|improve this answer









$endgroup$









  • 2




    $begingroup$
    This A052182 Your formula is identical to the last one given there, expect that the signs given there are incorrect. Perhaps you should publish a correction.
    $endgroup$
    – saulspatz
    Jan 7 at 21:46










  • $begingroup$
    It appears that the order of rows in that entry differs from the order of rows in this question (see the 3x3 matrix, for example). Therefore I believe the signs are correct in that list.
    $endgroup$
    – Klint Qinami
    Jan 8 at 1:00










  • $begingroup$
    You're right. Careless of me.
    $endgroup$
    – saulspatz
    Jan 8 at 3:10



















1












$begingroup$

These matrices are actually circulant matrices, whose determinant is well known.



For the matrices at hand, the determinant is
$
Delta_n=prod _{j=0}^{n-1}f(omega _{j})
$
,
where $f(z)=1+2z+3z^{2}+dots +nz^{n-1}$ and $w_j$ is the $j$-th root of unity.



We have $f(omega_0)=f(1)=1+2+3+cdots +n = dfrac{n(n+1)}{2}$.



We also have $f(z)=g'(z)$, where $g(z)=1+z+z^2+z^3+cdots+z^n=dfrac{z^{n+1}-1}{z-1}$. Therefore,
$$
g'(z)=dfrac{n z^{n+1} - (n+1) z^n + 1}{(z-1)^2}
$$

and so, for $jne0$,
$$
f(omega_j)=g'(omega_j)=dfrac{n omega_j - n}{(omega_j-1)^2}=dfrac{n}{omega_j-1}
$$

Thus,
$$
Delta_n=prod _{j=0}^{n-1}f(omega _{j})
=dfrac{n(n+1)}{2} prod _{j=1}^{n-1}dfrac{n}{omega_j-1}
=dfrac{n^n(n+1)}{2} dfrac{1}{prod _{j=1}^{n-1}(omega_j-1)}
$$

Now $prod _{j=1}^{n-1}(omega_j-1)=h(1)$ for
$$
h(z)=prod _{j=1}^{n-1}(omega_j-z)
=(-1)^{n-1}prod _{j=1}^{n-1}(z-omega_j)
=(-1)^{n-1}dfrac{z^{n}-1}{z-1}
=(-1)^{n-1}(1+z+z^2+z^3+cdots+z^{n-1})
$$

and so $h(1)=(-1)^{n-1}n$. Thus
$$
Delta_n
=dfrac{n^n(n+1)}{2} dfrac{1}{(-1)^{n-1}n}
=(-1)^{n-1}dfrac{n^{n-1}(n+1)}{2}
$$






share|cite|improve this answer











$endgroup$













    Your Answer





    StackExchange.ifUsing("editor", function () {
    return StackExchange.using("mathjaxEditing", function () {
    StackExchange.MarkdownEditor.creationCallbacks.add(function (editor, postfix) {
    StackExchange.mathjaxEditing.prepareWmdForMathJax(editor, postfix, [["$", "$"], ["\\(","\\)"]]);
    });
    });
    }, "mathjax-editing");

    StackExchange.ready(function() {
    var channelOptions = {
    tags: "".split(" "),
    id: "69"
    };
    initTagRenderer("".split(" "), "".split(" "), channelOptions);

    StackExchange.using("externalEditor", function() {
    // Have to fire editor after snippets, if snippets enabled
    if (StackExchange.settings.snippets.snippetsEnabled) {
    StackExchange.using("snippets", function() {
    createEditor();
    });
    }
    else {
    createEditor();
    }
    });

    function createEditor() {
    StackExchange.prepareEditor({
    heartbeatType: 'answer',
    autoActivateHeartbeat: false,
    convertImagesToLinks: true,
    noModals: true,
    showLowRepImageUploadWarning: true,
    reputationToPostImages: 10,
    bindNavPrevention: true,
    postfix: "",
    imageUploader: {
    brandingHtml: "Powered by u003ca class="icon-imgur-white" href="https://imgur.com/"u003eu003c/au003e",
    contentPolicyHtml: "User contributions licensed under u003ca href="https://creativecommons.org/licenses/by-sa/3.0/"u003ecc by-sa 3.0 with attribution requiredu003c/au003e u003ca href="https://stackoverflow.com/legal/content-policy"u003e(content policy)u003c/au003e",
    allowUrls: true
    },
    noCode: true, onDemand: true,
    discardSelector: ".discard-answer"
    ,immediatelyShowMarkdownHelp:true
    });


    }
    });














    draft saved

    draft discarded


















    StackExchange.ready(
    function () {
    StackExchange.openid.initPostLogin('.new-post-login', 'https%3a%2f%2fmath.stackexchange.com%2fquestions%2f3065461%2fdeterminant-of-a-hankel-matrix-with-sequence-1-2-3-n-1-2-n-1%23new-answer', 'question_page');
    }
    );

    Post as a guest















    Required, but never shown

























    2 Answers
    2






    active

    oldest

    votes








    2 Answers
    2






    active

    oldest

    votes









    active

    oldest

    votes






    active

    oldest

    votes









    2












    $begingroup$

    The magnitude of the determinant is $$frac{n^{n-1}(n+1)}{2}.$$ The sign is given by $$f(n) = begin{cases}+ & mathrm{floor}(n/2) text{ is even.} \ - & mathrm{floor}(n/2) text{ is odd} .end{cases}$$A useful formula is $sum_{i=1}^{n} i = frac{n(n+1)}{2}$.






    share|cite|improve this answer









    $endgroup$









    • 2




      $begingroup$
      This A052182 Your formula is identical to the last one given there, expect that the signs given there are incorrect. Perhaps you should publish a correction.
      $endgroup$
      – saulspatz
      Jan 7 at 21:46










    • $begingroup$
      It appears that the order of rows in that entry differs from the order of rows in this question (see the 3x3 matrix, for example). Therefore I believe the signs are correct in that list.
      $endgroup$
      – Klint Qinami
      Jan 8 at 1:00










    • $begingroup$
      You're right. Careless of me.
      $endgroup$
      – saulspatz
      Jan 8 at 3:10
















    2












    $begingroup$

    The magnitude of the determinant is $$frac{n^{n-1}(n+1)}{2}.$$ The sign is given by $$f(n) = begin{cases}+ & mathrm{floor}(n/2) text{ is even.} \ - & mathrm{floor}(n/2) text{ is odd} .end{cases}$$A useful formula is $sum_{i=1}^{n} i = frac{n(n+1)}{2}$.






    share|cite|improve this answer









    $endgroup$









    • 2




      $begingroup$
      This A052182 Your formula is identical to the last one given there, expect that the signs given there are incorrect. Perhaps you should publish a correction.
      $endgroup$
      – saulspatz
      Jan 7 at 21:46










    • $begingroup$
      It appears that the order of rows in that entry differs from the order of rows in this question (see the 3x3 matrix, for example). Therefore I believe the signs are correct in that list.
      $endgroup$
      – Klint Qinami
      Jan 8 at 1:00










    • $begingroup$
      You're right. Careless of me.
      $endgroup$
      – saulspatz
      Jan 8 at 3:10














    2












    2








    2





    $begingroup$

    The magnitude of the determinant is $$frac{n^{n-1}(n+1)}{2}.$$ The sign is given by $$f(n) = begin{cases}+ & mathrm{floor}(n/2) text{ is even.} \ - & mathrm{floor}(n/2) text{ is odd} .end{cases}$$A useful formula is $sum_{i=1}^{n} i = frac{n(n+1)}{2}$.






    share|cite|improve this answer









    $endgroup$



    The magnitude of the determinant is $$frac{n^{n-1}(n+1)}{2}.$$ The sign is given by $$f(n) = begin{cases}+ & mathrm{floor}(n/2) text{ is even.} \ - & mathrm{floor}(n/2) text{ is odd} .end{cases}$$A useful formula is $sum_{i=1}^{n} i = frac{n(n+1)}{2}$.







    share|cite|improve this answer












    share|cite|improve this answer



    share|cite|improve this answer










    answered Jan 7 at 21:27









    Klint QinamiKlint Qinami

    1,137410




    1,137410








    • 2




      $begingroup$
      This A052182 Your formula is identical to the last one given there, expect that the signs given there are incorrect. Perhaps you should publish a correction.
      $endgroup$
      – saulspatz
      Jan 7 at 21:46










    • $begingroup$
      It appears that the order of rows in that entry differs from the order of rows in this question (see the 3x3 matrix, for example). Therefore I believe the signs are correct in that list.
      $endgroup$
      – Klint Qinami
      Jan 8 at 1:00










    • $begingroup$
      You're right. Careless of me.
      $endgroup$
      – saulspatz
      Jan 8 at 3:10














    • 2




      $begingroup$
      This A052182 Your formula is identical to the last one given there, expect that the signs given there are incorrect. Perhaps you should publish a correction.
      $endgroup$
      – saulspatz
      Jan 7 at 21:46










    • $begingroup$
      It appears that the order of rows in that entry differs from the order of rows in this question (see the 3x3 matrix, for example). Therefore I believe the signs are correct in that list.
      $endgroup$
      – Klint Qinami
      Jan 8 at 1:00










    • $begingroup$
      You're right. Careless of me.
      $endgroup$
      – saulspatz
      Jan 8 at 3:10








    2




    2




    $begingroup$
    This A052182 Your formula is identical to the last one given there, expect that the signs given there are incorrect. Perhaps you should publish a correction.
    $endgroup$
    – saulspatz
    Jan 7 at 21:46




    $begingroup$
    This A052182 Your formula is identical to the last one given there, expect that the signs given there are incorrect. Perhaps you should publish a correction.
    $endgroup$
    – saulspatz
    Jan 7 at 21:46












    $begingroup$
    It appears that the order of rows in that entry differs from the order of rows in this question (see the 3x3 matrix, for example). Therefore I believe the signs are correct in that list.
    $endgroup$
    – Klint Qinami
    Jan 8 at 1:00




    $begingroup$
    It appears that the order of rows in that entry differs from the order of rows in this question (see the 3x3 matrix, for example). Therefore I believe the signs are correct in that list.
    $endgroup$
    – Klint Qinami
    Jan 8 at 1:00












    $begingroup$
    You're right. Careless of me.
    $endgroup$
    – saulspatz
    Jan 8 at 3:10




    $begingroup$
    You're right. Careless of me.
    $endgroup$
    – saulspatz
    Jan 8 at 3:10











    1












    $begingroup$

    These matrices are actually circulant matrices, whose determinant is well known.



    For the matrices at hand, the determinant is
    $
    Delta_n=prod _{j=0}^{n-1}f(omega _{j})
    $
    ,
    where $f(z)=1+2z+3z^{2}+dots +nz^{n-1}$ and $w_j$ is the $j$-th root of unity.



    We have $f(omega_0)=f(1)=1+2+3+cdots +n = dfrac{n(n+1)}{2}$.



    We also have $f(z)=g'(z)$, where $g(z)=1+z+z^2+z^3+cdots+z^n=dfrac{z^{n+1}-1}{z-1}$. Therefore,
    $$
    g'(z)=dfrac{n z^{n+1} - (n+1) z^n + 1}{(z-1)^2}
    $$

    and so, for $jne0$,
    $$
    f(omega_j)=g'(omega_j)=dfrac{n omega_j - n}{(omega_j-1)^2}=dfrac{n}{omega_j-1}
    $$

    Thus,
    $$
    Delta_n=prod _{j=0}^{n-1}f(omega _{j})
    =dfrac{n(n+1)}{2} prod _{j=1}^{n-1}dfrac{n}{omega_j-1}
    =dfrac{n^n(n+1)}{2} dfrac{1}{prod _{j=1}^{n-1}(omega_j-1)}
    $$

    Now $prod _{j=1}^{n-1}(omega_j-1)=h(1)$ for
    $$
    h(z)=prod _{j=1}^{n-1}(omega_j-z)
    =(-1)^{n-1}prod _{j=1}^{n-1}(z-omega_j)
    =(-1)^{n-1}dfrac{z^{n}-1}{z-1}
    =(-1)^{n-1}(1+z+z^2+z^3+cdots+z^{n-1})
    $$

    and so $h(1)=(-1)^{n-1}n$. Thus
    $$
    Delta_n
    =dfrac{n^n(n+1)}{2} dfrac{1}{(-1)^{n-1}n}
    =(-1)^{n-1}dfrac{n^{n-1}(n+1)}{2}
    $$






    share|cite|improve this answer











    $endgroup$


















      1












      $begingroup$

      These matrices are actually circulant matrices, whose determinant is well known.



      For the matrices at hand, the determinant is
      $
      Delta_n=prod _{j=0}^{n-1}f(omega _{j})
      $
      ,
      where $f(z)=1+2z+3z^{2}+dots +nz^{n-1}$ and $w_j$ is the $j$-th root of unity.



      We have $f(omega_0)=f(1)=1+2+3+cdots +n = dfrac{n(n+1)}{2}$.



      We also have $f(z)=g'(z)$, where $g(z)=1+z+z^2+z^3+cdots+z^n=dfrac{z^{n+1}-1}{z-1}$. Therefore,
      $$
      g'(z)=dfrac{n z^{n+1} - (n+1) z^n + 1}{(z-1)^2}
      $$

      and so, for $jne0$,
      $$
      f(omega_j)=g'(omega_j)=dfrac{n omega_j - n}{(omega_j-1)^2}=dfrac{n}{omega_j-1}
      $$

      Thus,
      $$
      Delta_n=prod _{j=0}^{n-1}f(omega _{j})
      =dfrac{n(n+1)}{2} prod _{j=1}^{n-1}dfrac{n}{omega_j-1}
      =dfrac{n^n(n+1)}{2} dfrac{1}{prod _{j=1}^{n-1}(omega_j-1)}
      $$

      Now $prod _{j=1}^{n-1}(omega_j-1)=h(1)$ for
      $$
      h(z)=prod _{j=1}^{n-1}(omega_j-z)
      =(-1)^{n-1}prod _{j=1}^{n-1}(z-omega_j)
      =(-1)^{n-1}dfrac{z^{n}-1}{z-1}
      =(-1)^{n-1}(1+z+z^2+z^3+cdots+z^{n-1})
      $$

      and so $h(1)=(-1)^{n-1}n$. Thus
      $$
      Delta_n
      =dfrac{n^n(n+1)}{2} dfrac{1}{(-1)^{n-1}n}
      =(-1)^{n-1}dfrac{n^{n-1}(n+1)}{2}
      $$






      share|cite|improve this answer











      $endgroup$
















        1












        1








        1





        $begingroup$

        These matrices are actually circulant matrices, whose determinant is well known.



        For the matrices at hand, the determinant is
        $
        Delta_n=prod _{j=0}^{n-1}f(omega _{j})
        $
        ,
        where $f(z)=1+2z+3z^{2}+dots +nz^{n-1}$ and $w_j$ is the $j$-th root of unity.



        We have $f(omega_0)=f(1)=1+2+3+cdots +n = dfrac{n(n+1)}{2}$.



        We also have $f(z)=g'(z)$, where $g(z)=1+z+z^2+z^3+cdots+z^n=dfrac{z^{n+1}-1}{z-1}$. Therefore,
        $$
        g'(z)=dfrac{n z^{n+1} - (n+1) z^n + 1}{(z-1)^2}
        $$

        and so, for $jne0$,
        $$
        f(omega_j)=g'(omega_j)=dfrac{n omega_j - n}{(omega_j-1)^2}=dfrac{n}{omega_j-1}
        $$

        Thus,
        $$
        Delta_n=prod _{j=0}^{n-1}f(omega _{j})
        =dfrac{n(n+1)}{2} prod _{j=1}^{n-1}dfrac{n}{omega_j-1}
        =dfrac{n^n(n+1)}{2} dfrac{1}{prod _{j=1}^{n-1}(omega_j-1)}
        $$

        Now $prod _{j=1}^{n-1}(omega_j-1)=h(1)$ for
        $$
        h(z)=prod _{j=1}^{n-1}(omega_j-z)
        =(-1)^{n-1}prod _{j=1}^{n-1}(z-omega_j)
        =(-1)^{n-1}dfrac{z^{n}-1}{z-1}
        =(-1)^{n-1}(1+z+z^2+z^3+cdots+z^{n-1})
        $$

        and so $h(1)=(-1)^{n-1}n$. Thus
        $$
        Delta_n
        =dfrac{n^n(n+1)}{2} dfrac{1}{(-1)^{n-1}n}
        =(-1)^{n-1}dfrac{n^{n-1}(n+1)}{2}
        $$






        share|cite|improve this answer











        $endgroup$



        These matrices are actually circulant matrices, whose determinant is well known.



        For the matrices at hand, the determinant is
        $
        Delta_n=prod _{j=0}^{n-1}f(omega _{j})
        $
        ,
        where $f(z)=1+2z+3z^{2}+dots +nz^{n-1}$ and $w_j$ is the $j$-th root of unity.



        We have $f(omega_0)=f(1)=1+2+3+cdots +n = dfrac{n(n+1)}{2}$.



        We also have $f(z)=g'(z)$, where $g(z)=1+z+z^2+z^3+cdots+z^n=dfrac{z^{n+1}-1}{z-1}$. Therefore,
        $$
        g'(z)=dfrac{n z^{n+1} - (n+1) z^n + 1}{(z-1)^2}
        $$

        and so, for $jne0$,
        $$
        f(omega_j)=g'(omega_j)=dfrac{n omega_j - n}{(omega_j-1)^2}=dfrac{n}{omega_j-1}
        $$

        Thus,
        $$
        Delta_n=prod _{j=0}^{n-1}f(omega _{j})
        =dfrac{n(n+1)}{2} prod _{j=1}^{n-1}dfrac{n}{omega_j-1}
        =dfrac{n^n(n+1)}{2} dfrac{1}{prod _{j=1}^{n-1}(omega_j-1)}
        $$

        Now $prod _{j=1}^{n-1}(omega_j-1)=h(1)$ for
        $$
        h(z)=prod _{j=1}^{n-1}(omega_j-z)
        =(-1)^{n-1}prod _{j=1}^{n-1}(z-omega_j)
        =(-1)^{n-1}dfrac{z^{n}-1}{z-1}
        =(-1)^{n-1}(1+z+z^2+z^3+cdots+z^{n-1})
        $$

        and so $h(1)=(-1)^{n-1}n$. Thus
        $$
        Delta_n
        =dfrac{n^n(n+1)}{2} dfrac{1}{(-1)^{n-1}n}
        =(-1)^{n-1}dfrac{n^{n-1}(n+1)}{2}
        $$







        share|cite|improve this answer














        share|cite|improve this answer



        share|cite|improve this answer








        edited Jan 8 at 13:00

























        answered Jan 8 at 11:27









        lhflhf

        163k10169393




        163k10169393






























            draft saved

            draft discarded




















































            Thanks for contributing an answer to Mathematics Stack Exchange!


            • Please be sure to answer the question. Provide details and share your research!

            But avoid



            • Asking for help, clarification, or responding to other answers.

            • Making statements based on opinion; back them up with references or personal experience.


            Use MathJax to format equations. MathJax reference.


            To learn more, see our tips on writing great answers.




            draft saved


            draft discarded














            StackExchange.ready(
            function () {
            StackExchange.openid.initPostLogin('.new-post-login', 'https%3a%2f%2fmath.stackexchange.com%2fquestions%2f3065461%2fdeterminant-of-a-hankel-matrix-with-sequence-1-2-3-n-1-2-n-1%23new-answer', 'question_page');
            }
            );

            Post as a guest















            Required, but never shown





















































            Required, but never shown














            Required, but never shown












            Required, but never shown







            Required, but never shown

































            Required, but never shown














            Required, but never shown












            Required, but never shown







            Required, but never shown







            Popular posts from this blog

            An IMO inspired problem

            Management

            Investment